42c3117099743ee436269c13817e2da8af54e681
[course.git] / latex / problems / Serway_and_Jewett_8 / problem06.08.tex
1 \begin{problem*}{6.8}
2 Consider a conical pendulum (Fig.~P6.8) with a bob of mass
3 $m=80.0\U{kg}$ on a string of length $L=10.0\U{m}$ that makes an angle
4 of $\theta=5.00\dg$ with the vertical.  Determine \Part{a} the
5 horizontal and vertical components of the force exerted by the string
6 on the pendulum and \Part{b} the radial acceleration of the bob.
7 \begin{center}
8 \begin{asy}
9 import three;
10
11 real u = 1cm;
12
13 draw((0,0,2u)--(0,0,0), dashed);
14 draw((0,0,2u)--(0,u,0));
15 draw(scale3(u)*unitcircle3);
16 dot((0,0,2u));
17 dot((0,u,0));
18 \end{asy}
19 \end{center}
20 \end{problem*}
21
22 \begin{solution}
23 \Part{a}
24 Drawing a free body diagram for the bob
25 \begin{center}
26 \begin{asy}
27 import Mechanics;
28
29 real u = 1cm;
30 real theta = 25;
31 real mg = 1.3u;
32
33 draw((0,0)--(0, mg));
34 Angle t = Angle(dir(90), (0,0), dir(90+theta), "$\theta$");
35 t.draw();
36 Vector T = Force((0,0), mag=mg/Cos(theta), dir=(90+theta), "$T$");
37 T.draw();
38 Vector G = Force((0,0), mag=mg, dir=(-90), "$mg$");
39 G.draw();
40 dot((0,0));
41
42 draw_ijhat((.5u,0), idir=0);
43 \end{asy}
44 \end{center}
45
46 The bob does not move up or down, so the sum of forces in the vertical
47 direction must be zero.
48 \begin{align}
49   0 &= \sum F_y = T_y - mg \\
50   T\cos(\theta) = T_y &= mg
51     = 80.0\U{kg}\cdot9.80\U{m/s$^2$} = \ans{784\U{N}} \\
52   T &= \frac{mg}{\cos(\theta)} \;,
53 \end{align}
54 where I've marked the vertical component of the string tension.  The
55 horizontal (radial) component is given by
56 \begin{equation}
57   T_x = -T\sin(\theta)
58     = -\frac{mg\sin(\theta)}{\cos(\theta)}
59     = -mg\tan(\theta) = \ans{-68.6\U{N}} \;.
60 \end{equation}
61
62 \Part{b}
63 The radial acceleration is given by Newton's second law
64 \begin{align}
65   F_r &= m a_r \\
66   a_r &= \frac{F_r}{m} = \frac{T_x}{m}
67     = \frac{-mg\tan(\theta)}{m}
68     = -g\tan(\theta)
69     = -9.80\U{m/s$^2$}\cdot\tan(5.00\dg)
70     = \ans{0.857\U{m/s$^2$}} \;.
71 \end{align}
72 \end{solution}